Problem from Dixon&Mortimer textbook: (allegedly) sufficient conditions for a transporter between two finite subsets to be non-emptySufficient conditions for ultracharactericitySpecify finite group satisfying two conditionsSufficient conditions for $Gcong Ntimes G/N$$AGL(V) = V rtimes GL(V)$ with $GL(V)$ acting from the rightSufficient Conditions for the Commutator Subgroup to be a ComponentSufficient Conditions for $(R,+,.)$ to be a commutative ringNecessary and Sufficient Conditions for Subgroup InclusionsSome conditions on a finite non-abelian $2$-groupSufficient conditions for a group homomorphism/isomorphismSufficient conditions on the isomorphism of two groups

I am confused as to how the inverse of a certain function is found.

Custom alignment for GeoMarkers

What are substitutions for coconut in curry?

Can I use USB data pins as a power source?

Knife as defense against stray dogs

How could a scammer know the apps on my phone / iTunes account?

How do I hide Chekhov's Gun?

Aluminum electrolytic or ceramic capacitors for linear regulator input and output?

How do you talk to someone whose loved one is dying?

What is the significance behind "40 days" that often appears in the Bible?

Is it normal that my co-workers at a fitness company criticize my food choices?

How to write cleanly even if my character uses expletive language?

This word with a lot of past tenses

I got the following comment from a reputed math journal. What does it mean?

A single argument pattern definition applies to multiple-argument patterns?

Why do tuner card drivers fail to build after kernel update to 4.4.0-143-generic?

Describing a chess game in a novel

PTIJ: Who should I vote for? (21st Knesset Edition)

What does 高層ビルに何車線もの道路。mean?

Non-trivial topology where only open sets are closed

Is there a symmetric-key algorithm which we can use for creating a signature?

Instead of a Universal Basic Income program, why not implement a "Universal Basic Needs" program?

How should I state my peer review experience in the CV?

What exactly is this small puffer fish doing and how did it manage to accomplish such a feat?



Problem from Dixon&Mortimer textbook: (allegedly) sufficient conditions for a transporter between two finite subsets to be non-empty


Sufficient conditions for ultracharactericitySpecify finite group satisfying two conditionsSufficient conditions for $Gcong Ntimes G/N$$AGL(V) = V rtimes GL(V)$ with $GL(V)$ acting from the rightSufficient Conditions for the Commutator Subgroup to be a ComponentSufficient Conditions for $(R,+,.)$ to be a commutative ringNecessary and Sufficient Conditions for Subgroup InclusionsSome conditions on a finite non-abelian $2$-groupSufficient conditions for a group homomorphism/isomorphismSufficient conditions on the isomorphism of two groups













2












$begingroup$


I should perhaps begin by apologizing for asking a not very profound question, but there is something that bothers me about exercise 1.5.19 from Permutation groups by Dixon&Mortimer. May I restate the hypothesis in a language closer to my personal preference. For any action of a group $G$ on a set $A$ and any subset $Xsubseteq A$ define its fixator as $$mathrmFix_G X=bigcap_x in X mathrmStab_G x$$
when $X neq emptyset$ and set in particular $mathrmFix_G emptyset=G$.
Assume now the following for a given transitive action of $G$ on $A$:




  1. $B, C subseteq A$ are finite such that $|B| leqslant |C|$


  2. $mathrmFix_G B$ and $mathrmFix_G C$ act transitively
    on the complementary subsets $A setminus B$ respectively $Asetminus C$.

Then the exercise would have it that there exists an operator $lambda in G$ such that $lambda B subseteq C$. The exercise also asks whether the alleged result remains valid for infinite subsets $B$ and $C$.



Although I can't come up right away with a counterexample, these hypotheses of mere transitivity seem too weak to afford the nonemptiness of the transporter of $B$ to $C$. Expressing the conditions of the problem in terms of a certain point stabilizer and intersections of some of its conjugates (which is what the fixators would amount to) also doesn't seem too useful. There will exist of course only finitely many injections from $B$ to $C$, however why should one of them act like one of the operators of $G$ on $B$?



I can't completely deny the possibility of there being some trickery of maneuvering $B$ into $C$, but at any rate I can't see how. If anyone could enlighten me, I would be grateful.










share|cite|improve this question









$endgroup$







  • 1




    $begingroup$
    Move as much as you can of $B$ into $C$, and suppose there is still some $b$ in $B$ that you have not managed to move into $C$. Use an element in the fixator of $C$ to move $b$ out of $B$ and then use an element in the fixator of $B$ to move $b$ from its current positiuon outside of $B$ into $C$.
    $endgroup$
    – Derek Holt
    Mar 12 at 9:57











  • $begingroup$
    Thank you kindly for your reply! There is just one detail that needs to be ascertained in order to carry out the argument as you suggested it, namely that $Bcup C neq A$. I am not entirely sure whether the conditions of the problem would preclude this instance.
    $endgroup$
    – ΑΘΩ
    Mar 12 at 11:48











  • $begingroup$
    Yes you are right that is a problem - I will think some more.
    $endgroup$
    – Derek Holt
    Mar 12 at 12:06










  • $begingroup$
    Another observation right off the bat is that apart from the ''singular instance'' which requires separate treatment, the hypothesis of finiteness on $C$ was unnecessary. It may play a role in preventing the ''singular'' instance.
    $endgroup$
    – ΑΘΩ
    Mar 12 at 12:08










  • $begingroup$
    But the result is false when $B$ is allowed to be infinite. You can have silly counterexamples like $C=A$ and $B = A setminus a$.
    $endgroup$
    – Derek Holt
    Mar 12 at 12:49















2












$begingroup$


I should perhaps begin by apologizing for asking a not very profound question, but there is something that bothers me about exercise 1.5.19 from Permutation groups by Dixon&Mortimer. May I restate the hypothesis in a language closer to my personal preference. For any action of a group $G$ on a set $A$ and any subset $Xsubseteq A$ define its fixator as $$mathrmFix_G X=bigcap_x in X mathrmStab_G x$$
when $X neq emptyset$ and set in particular $mathrmFix_G emptyset=G$.
Assume now the following for a given transitive action of $G$ on $A$:




  1. $B, C subseteq A$ are finite such that $|B| leqslant |C|$


  2. $mathrmFix_G B$ and $mathrmFix_G C$ act transitively
    on the complementary subsets $A setminus B$ respectively $Asetminus C$.

Then the exercise would have it that there exists an operator $lambda in G$ such that $lambda B subseteq C$. The exercise also asks whether the alleged result remains valid for infinite subsets $B$ and $C$.



Although I can't come up right away with a counterexample, these hypotheses of mere transitivity seem too weak to afford the nonemptiness of the transporter of $B$ to $C$. Expressing the conditions of the problem in terms of a certain point stabilizer and intersections of some of its conjugates (which is what the fixators would amount to) also doesn't seem too useful. There will exist of course only finitely many injections from $B$ to $C$, however why should one of them act like one of the operators of $G$ on $B$?



I can't completely deny the possibility of there being some trickery of maneuvering $B$ into $C$, but at any rate I can't see how. If anyone could enlighten me, I would be grateful.










share|cite|improve this question









$endgroup$







  • 1




    $begingroup$
    Move as much as you can of $B$ into $C$, and suppose there is still some $b$ in $B$ that you have not managed to move into $C$. Use an element in the fixator of $C$ to move $b$ out of $B$ and then use an element in the fixator of $B$ to move $b$ from its current positiuon outside of $B$ into $C$.
    $endgroup$
    – Derek Holt
    Mar 12 at 9:57











  • $begingroup$
    Thank you kindly for your reply! There is just one detail that needs to be ascertained in order to carry out the argument as you suggested it, namely that $Bcup C neq A$. I am not entirely sure whether the conditions of the problem would preclude this instance.
    $endgroup$
    – ΑΘΩ
    Mar 12 at 11:48











  • $begingroup$
    Yes you are right that is a problem - I will think some more.
    $endgroup$
    – Derek Holt
    Mar 12 at 12:06










  • $begingroup$
    Another observation right off the bat is that apart from the ''singular instance'' which requires separate treatment, the hypothesis of finiteness on $C$ was unnecessary. It may play a role in preventing the ''singular'' instance.
    $endgroup$
    – ΑΘΩ
    Mar 12 at 12:08










  • $begingroup$
    But the result is false when $B$ is allowed to be infinite. You can have silly counterexamples like $C=A$ and $B = A setminus a$.
    $endgroup$
    – Derek Holt
    Mar 12 at 12:49













2












2








2





$begingroup$


I should perhaps begin by apologizing for asking a not very profound question, but there is something that bothers me about exercise 1.5.19 from Permutation groups by Dixon&Mortimer. May I restate the hypothesis in a language closer to my personal preference. For any action of a group $G$ on a set $A$ and any subset $Xsubseteq A$ define its fixator as $$mathrmFix_G X=bigcap_x in X mathrmStab_G x$$
when $X neq emptyset$ and set in particular $mathrmFix_G emptyset=G$.
Assume now the following for a given transitive action of $G$ on $A$:




  1. $B, C subseteq A$ are finite such that $|B| leqslant |C|$


  2. $mathrmFix_G B$ and $mathrmFix_G C$ act transitively
    on the complementary subsets $A setminus B$ respectively $Asetminus C$.

Then the exercise would have it that there exists an operator $lambda in G$ such that $lambda B subseteq C$. The exercise also asks whether the alleged result remains valid for infinite subsets $B$ and $C$.



Although I can't come up right away with a counterexample, these hypotheses of mere transitivity seem too weak to afford the nonemptiness of the transporter of $B$ to $C$. Expressing the conditions of the problem in terms of a certain point stabilizer and intersections of some of its conjugates (which is what the fixators would amount to) also doesn't seem too useful. There will exist of course only finitely many injections from $B$ to $C$, however why should one of them act like one of the operators of $G$ on $B$?



I can't completely deny the possibility of there being some trickery of maneuvering $B$ into $C$, but at any rate I can't see how. If anyone could enlighten me, I would be grateful.










share|cite|improve this question









$endgroup$




I should perhaps begin by apologizing for asking a not very profound question, but there is something that bothers me about exercise 1.5.19 from Permutation groups by Dixon&Mortimer. May I restate the hypothesis in a language closer to my personal preference. For any action of a group $G$ on a set $A$ and any subset $Xsubseteq A$ define its fixator as $$mathrmFix_G X=bigcap_x in X mathrmStab_G x$$
when $X neq emptyset$ and set in particular $mathrmFix_G emptyset=G$.
Assume now the following for a given transitive action of $G$ on $A$:




  1. $B, C subseteq A$ are finite such that $|B| leqslant |C|$


  2. $mathrmFix_G B$ and $mathrmFix_G C$ act transitively
    on the complementary subsets $A setminus B$ respectively $Asetminus C$.

Then the exercise would have it that there exists an operator $lambda in G$ such that $lambda B subseteq C$. The exercise also asks whether the alleged result remains valid for infinite subsets $B$ and $C$.



Although I can't come up right away with a counterexample, these hypotheses of mere transitivity seem too weak to afford the nonemptiness of the transporter of $B$ to $C$. Expressing the conditions of the problem in terms of a certain point stabilizer and intersections of some of its conjugates (which is what the fixators would amount to) also doesn't seem too useful. There will exist of course only finitely many injections from $B$ to $C$, however why should one of them act like one of the operators of $G$ on $B$?



I can't completely deny the possibility of there being some trickery of maneuvering $B$ into $C$, but at any rate I can't see how. If anyone could enlighten me, I would be grateful.







group-theory






share|cite|improve this question













share|cite|improve this question











share|cite|improve this question




share|cite|improve this question










asked Mar 12 at 9:49









ΑΘΩΑΘΩ

3015




3015







  • 1




    $begingroup$
    Move as much as you can of $B$ into $C$, and suppose there is still some $b$ in $B$ that you have not managed to move into $C$. Use an element in the fixator of $C$ to move $b$ out of $B$ and then use an element in the fixator of $B$ to move $b$ from its current positiuon outside of $B$ into $C$.
    $endgroup$
    – Derek Holt
    Mar 12 at 9:57











  • $begingroup$
    Thank you kindly for your reply! There is just one detail that needs to be ascertained in order to carry out the argument as you suggested it, namely that $Bcup C neq A$. I am not entirely sure whether the conditions of the problem would preclude this instance.
    $endgroup$
    – ΑΘΩ
    Mar 12 at 11:48











  • $begingroup$
    Yes you are right that is a problem - I will think some more.
    $endgroup$
    – Derek Holt
    Mar 12 at 12:06










  • $begingroup$
    Another observation right off the bat is that apart from the ''singular instance'' which requires separate treatment, the hypothesis of finiteness on $C$ was unnecessary. It may play a role in preventing the ''singular'' instance.
    $endgroup$
    – ΑΘΩ
    Mar 12 at 12:08










  • $begingroup$
    But the result is false when $B$ is allowed to be infinite. You can have silly counterexamples like $C=A$ and $B = A setminus a$.
    $endgroup$
    – Derek Holt
    Mar 12 at 12:49












  • 1




    $begingroup$
    Move as much as you can of $B$ into $C$, and suppose there is still some $b$ in $B$ that you have not managed to move into $C$. Use an element in the fixator of $C$ to move $b$ out of $B$ and then use an element in the fixator of $B$ to move $b$ from its current positiuon outside of $B$ into $C$.
    $endgroup$
    – Derek Holt
    Mar 12 at 9:57











  • $begingroup$
    Thank you kindly for your reply! There is just one detail that needs to be ascertained in order to carry out the argument as you suggested it, namely that $Bcup C neq A$. I am not entirely sure whether the conditions of the problem would preclude this instance.
    $endgroup$
    – ΑΘΩ
    Mar 12 at 11:48











  • $begingroup$
    Yes you are right that is a problem - I will think some more.
    $endgroup$
    – Derek Holt
    Mar 12 at 12:06










  • $begingroup$
    Another observation right off the bat is that apart from the ''singular instance'' which requires separate treatment, the hypothesis of finiteness on $C$ was unnecessary. It may play a role in preventing the ''singular'' instance.
    $endgroup$
    – ΑΘΩ
    Mar 12 at 12:08










  • $begingroup$
    But the result is false when $B$ is allowed to be infinite. You can have silly counterexamples like $C=A$ and $B = A setminus a$.
    $endgroup$
    – Derek Holt
    Mar 12 at 12:49







1




1




$begingroup$
Move as much as you can of $B$ into $C$, and suppose there is still some $b$ in $B$ that you have not managed to move into $C$. Use an element in the fixator of $C$ to move $b$ out of $B$ and then use an element in the fixator of $B$ to move $b$ from its current positiuon outside of $B$ into $C$.
$endgroup$
– Derek Holt
Mar 12 at 9:57





$begingroup$
Move as much as you can of $B$ into $C$, and suppose there is still some $b$ in $B$ that you have not managed to move into $C$. Use an element in the fixator of $C$ to move $b$ out of $B$ and then use an element in the fixator of $B$ to move $b$ from its current positiuon outside of $B$ into $C$.
$endgroup$
– Derek Holt
Mar 12 at 9:57













$begingroup$
Thank you kindly for your reply! There is just one detail that needs to be ascertained in order to carry out the argument as you suggested it, namely that $Bcup C neq A$. I am not entirely sure whether the conditions of the problem would preclude this instance.
$endgroup$
– ΑΘΩ
Mar 12 at 11:48





$begingroup$
Thank you kindly for your reply! There is just one detail that needs to be ascertained in order to carry out the argument as you suggested it, namely that $Bcup C neq A$. I am not entirely sure whether the conditions of the problem would preclude this instance.
$endgroup$
– ΑΘΩ
Mar 12 at 11:48













$begingroup$
Yes you are right that is a problem - I will think some more.
$endgroup$
– Derek Holt
Mar 12 at 12:06




$begingroup$
Yes you are right that is a problem - I will think some more.
$endgroup$
– Derek Holt
Mar 12 at 12:06












$begingroup$
Another observation right off the bat is that apart from the ''singular instance'' which requires separate treatment, the hypothesis of finiteness on $C$ was unnecessary. It may play a role in preventing the ''singular'' instance.
$endgroup$
– ΑΘΩ
Mar 12 at 12:08




$begingroup$
Another observation right off the bat is that apart from the ''singular instance'' which requires separate treatment, the hypothesis of finiteness on $C$ was unnecessary. It may play a role in preventing the ''singular'' instance.
$endgroup$
– ΑΘΩ
Mar 12 at 12:08












$begingroup$
But the result is false when $B$ is allowed to be infinite. You can have silly counterexamples like $C=A$ and $B = A setminus a$.
$endgroup$
– Derek Holt
Mar 12 at 12:49




$begingroup$
But the result is false when $B$ is allowed to be infinite. You can have silly counterexamples like $C=A$ and $B = A setminus a$.
$endgroup$
– Derek Holt
Mar 12 at 12:49










1 Answer
1






active

oldest

votes


















0












$begingroup$

I prefer to use Dixon&Mortimer's notation $G_(X)$ for what you call the fixator of $X$ in $G$. And like D&M, I very much prefer to use right actions, so I will use $B^g$ where you write $gB$.



Choose $g in G$ such that $|B^g cap C|$ is as large as possible, and then replace $B$ with $B^g$. If $B cap C = B$ then we are done so suppose not. So there exists $b in B setminus C$. Since $|C| ge |B|$ and $B,C$ are finite, there exists $c in C setminus B$.



If $A ne B cap C$, then there exists $a in A setminus (B cap C)$, and we can find $g in G_(C)$ with $b^g = a$ and $h in G_(B)$ with $a^h=c$, so $b^gh = c$, and since $(B cap C)^gh = B cap C$, we now have $(B cap C) cup c in B^gh cap C$, contrary to the maximality of $|B^g cap C|$.



It remains to deal with the case when $A = B cup C$, in which canse $A$ is the disjoint union of $B cap C$, $B setminus C$ and $C setminus B$ (so in particular $A$ is finite). By transitivity of $G$ on $A$ (which we haven't used yet), there exsists $g in G$ with $c^g = b$. Since $c notin B$, $g$ can map at most $|B setminus C| - 1$ elements of $B$ into $B setminus C = A setminus C$. So $g$ must map at least $|B| - |B setminus C| + 1 = |B cap C| + 1$ elements of $B$ into $C$. Hence $|B^g cap C| > |B cap C|$ again contradicting the maximality of $|B cap C|$.






share|cite|improve this answer









$endgroup$












  • $begingroup$
    Thank you again! Your noble perseverance has shone light on the matter once and for all. As a remark, it seems indeed that the hypothesis that $C$ be finite is indeed superfluous (the only necessary aspect being the cardinal inequality between the two).
    $endgroup$
    – ΑΘΩ
    yesterday










Your Answer





StackExchange.ifUsing("editor", function ()
return StackExchange.using("mathjaxEditing", function ()
StackExchange.MarkdownEditor.creationCallbacks.add(function (editor, postfix)
StackExchange.mathjaxEditing.prepareWmdForMathJax(editor, postfix, [["$", "$"], ["\\(","\\)"]]);
);
);
, "mathjax-editing");

StackExchange.ready(function()
var channelOptions =
tags: "".split(" "),
id: "69"
;
initTagRenderer("".split(" "), "".split(" "), channelOptions);

StackExchange.using("externalEditor", function()
// Have to fire editor after snippets, if snippets enabled
if (StackExchange.settings.snippets.snippetsEnabled)
StackExchange.using("snippets", function()
createEditor();
);

else
createEditor();

);

function createEditor()
StackExchange.prepareEditor(
heartbeatType: 'answer',
autoActivateHeartbeat: false,
convertImagesToLinks: true,
noModals: true,
showLowRepImageUploadWarning: true,
reputationToPostImages: 10,
bindNavPrevention: true,
postfix: "",
imageUploader:
brandingHtml: "Powered by u003ca class="icon-imgur-white" href="https://imgur.com/"u003eu003c/au003e",
contentPolicyHtml: "User contributions licensed under u003ca href="https://creativecommons.org/licenses/by-sa/3.0/"u003ecc by-sa 3.0 with attribution requiredu003c/au003e u003ca href="https://stackoverflow.com/legal/content-policy"u003e(content policy)u003c/au003e",
allowUrls: true
,
noCode: true, onDemand: true,
discardSelector: ".discard-answer"
,immediatelyShowMarkdownHelp:true
);



);













draft saved

draft discarded


















StackExchange.ready(
function ()
StackExchange.openid.initPostLogin('.new-post-login', 'https%3a%2f%2fmath.stackexchange.com%2fquestions%2f3144880%2fproblem-from-dixonmortimer-textbook-allegedly-sufficient-conditions-for-a-tr%23new-answer', 'question_page');

);

Post as a guest















Required, but never shown

























1 Answer
1






active

oldest

votes








1 Answer
1






active

oldest

votes









active

oldest

votes






active

oldest

votes









0












$begingroup$

I prefer to use Dixon&Mortimer's notation $G_(X)$ for what you call the fixator of $X$ in $G$. And like D&M, I very much prefer to use right actions, so I will use $B^g$ where you write $gB$.



Choose $g in G$ such that $|B^g cap C|$ is as large as possible, and then replace $B$ with $B^g$. If $B cap C = B$ then we are done so suppose not. So there exists $b in B setminus C$. Since $|C| ge |B|$ and $B,C$ are finite, there exists $c in C setminus B$.



If $A ne B cap C$, then there exists $a in A setminus (B cap C)$, and we can find $g in G_(C)$ with $b^g = a$ and $h in G_(B)$ with $a^h=c$, so $b^gh = c$, and since $(B cap C)^gh = B cap C$, we now have $(B cap C) cup c in B^gh cap C$, contrary to the maximality of $|B^g cap C|$.



It remains to deal with the case when $A = B cup C$, in which canse $A$ is the disjoint union of $B cap C$, $B setminus C$ and $C setminus B$ (so in particular $A$ is finite). By transitivity of $G$ on $A$ (which we haven't used yet), there exsists $g in G$ with $c^g = b$. Since $c notin B$, $g$ can map at most $|B setminus C| - 1$ elements of $B$ into $B setminus C = A setminus C$. So $g$ must map at least $|B| - |B setminus C| + 1 = |B cap C| + 1$ elements of $B$ into $C$. Hence $|B^g cap C| > |B cap C|$ again contradicting the maximality of $|B cap C|$.






share|cite|improve this answer









$endgroup$












  • $begingroup$
    Thank you again! Your noble perseverance has shone light on the matter once and for all. As a remark, it seems indeed that the hypothesis that $C$ be finite is indeed superfluous (the only necessary aspect being the cardinal inequality between the two).
    $endgroup$
    – ΑΘΩ
    yesterday















0












$begingroup$

I prefer to use Dixon&Mortimer's notation $G_(X)$ for what you call the fixator of $X$ in $G$. And like D&M, I very much prefer to use right actions, so I will use $B^g$ where you write $gB$.



Choose $g in G$ such that $|B^g cap C|$ is as large as possible, and then replace $B$ with $B^g$. If $B cap C = B$ then we are done so suppose not. So there exists $b in B setminus C$. Since $|C| ge |B|$ and $B,C$ are finite, there exists $c in C setminus B$.



If $A ne B cap C$, then there exists $a in A setminus (B cap C)$, and we can find $g in G_(C)$ with $b^g = a$ and $h in G_(B)$ with $a^h=c$, so $b^gh = c$, and since $(B cap C)^gh = B cap C$, we now have $(B cap C) cup c in B^gh cap C$, contrary to the maximality of $|B^g cap C|$.



It remains to deal with the case when $A = B cup C$, in which canse $A$ is the disjoint union of $B cap C$, $B setminus C$ and $C setminus B$ (so in particular $A$ is finite). By transitivity of $G$ on $A$ (which we haven't used yet), there exsists $g in G$ with $c^g = b$. Since $c notin B$, $g$ can map at most $|B setminus C| - 1$ elements of $B$ into $B setminus C = A setminus C$. So $g$ must map at least $|B| - |B setminus C| + 1 = |B cap C| + 1$ elements of $B$ into $C$. Hence $|B^g cap C| > |B cap C|$ again contradicting the maximality of $|B cap C|$.






share|cite|improve this answer









$endgroup$












  • $begingroup$
    Thank you again! Your noble perseverance has shone light on the matter once and for all. As a remark, it seems indeed that the hypothesis that $C$ be finite is indeed superfluous (the only necessary aspect being the cardinal inequality between the two).
    $endgroup$
    – ΑΘΩ
    yesterday













0












0








0





$begingroup$

I prefer to use Dixon&Mortimer's notation $G_(X)$ for what you call the fixator of $X$ in $G$. And like D&M, I very much prefer to use right actions, so I will use $B^g$ where you write $gB$.



Choose $g in G$ such that $|B^g cap C|$ is as large as possible, and then replace $B$ with $B^g$. If $B cap C = B$ then we are done so suppose not. So there exists $b in B setminus C$. Since $|C| ge |B|$ and $B,C$ are finite, there exists $c in C setminus B$.



If $A ne B cap C$, then there exists $a in A setminus (B cap C)$, and we can find $g in G_(C)$ with $b^g = a$ and $h in G_(B)$ with $a^h=c$, so $b^gh = c$, and since $(B cap C)^gh = B cap C$, we now have $(B cap C) cup c in B^gh cap C$, contrary to the maximality of $|B^g cap C|$.



It remains to deal with the case when $A = B cup C$, in which canse $A$ is the disjoint union of $B cap C$, $B setminus C$ and $C setminus B$ (so in particular $A$ is finite). By transitivity of $G$ on $A$ (which we haven't used yet), there exsists $g in G$ with $c^g = b$. Since $c notin B$, $g$ can map at most $|B setminus C| - 1$ elements of $B$ into $B setminus C = A setminus C$. So $g$ must map at least $|B| - |B setminus C| + 1 = |B cap C| + 1$ elements of $B$ into $C$. Hence $|B^g cap C| > |B cap C|$ again contradicting the maximality of $|B cap C|$.






share|cite|improve this answer









$endgroup$



I prefer to use Dixon&Mortimer's notation $G_(X)$ for what you call the fixator of $X$ in $G$. And like D&M, I very much prefer to use right actions, so I will use $B^g$ where you write $gB$.



Choose $g in G$ such that $|B^g cap C|$ is as large as possible, and then replace $B$ with $B^g$. If $B cap C = B$ then we are done so suppose not. So there exists $b in B setminus C$. Since $|C| ge |B|$ and $B,C$ are finite, there exists $c in C setminus B$.



If $A ne B cap C$, then there exists $a in A setminus (B cap C)$, and we can find $g in G_(C)$ with $b^g = a$ and $h in G_(B)$ with $a^h=c$, so $b^gh = c$, and since $(B cap C)^gh = B cap C$, we now have $(B cap C) cup c in B^gh cap C$, contrary to the maximality of $|B^g cap C|$.



It remains to deal with the case when $A = B cup C$, in which canse $A$ is the disjoint union of $B cap C$, $B setminus C$ and $C setminus B$ (so in particular $A$ is finite). By transitivity of $G$ on $A$ (which we haven't used yet), there exsists $g in G$ with $c^g = b$. Since $c notin B$, $g$ can map at most $|B setminus C| - 1$ elements of $B$ into $B setminus C = A setminus C$. So $g$ must map at least $|B| - |B setminus C| + 1 = |B cap C| + 1$ elements of $B$ into $C$. Hence $|B^g cap C| > |B cap C|$ again contradicting the maximality of $|B cap C|$.







share|cite|improve this answer












share|cite|improve this answer



share|cite|improve this answer










answered 2 days ago









Derek HoltDerek Holt

54.2k53573




54.2k53573











  • $begingroup$
    Thank you again! Your noble perseverance has shone light on the matter once and for all. As a remark, it seems indeed that the hypothesis that $C$ be finite is indeed superfluous (the only necessary aspect being the cardinal inequality between the two).
    $endgroup$
    – ΑΘΩ
    yesterday
















  • $begingroup$
    Thank you again! Your noble perseverance has shone light on the matter once and for all. As a remark, it seems indeed that the hypothesis that $C$ be finite is indeed superfluous (the only necessary aspect being the cardinal inequality between the two).
    $endgroup$
    – ΑΘΩ
    yesterday















$begingroup$
Thank you again! Your noble perseverance has shone light on the matter once and for all. As a remark, it seems indeed that the hypothesis that $C$ be finite is indeed superfluous (the only necessary aspect being the cardinal inequality between the two).
$endgroup$
– ΑΘΩ
yesterday




$begingroup$
Thank you again! Your noble perseverance has shone light on the matter once and for all. As a remark, it seems indeed that the hypothesis that $C$ be finite is indeed superfluous (the only necessary aspect being the cardinal inequality between the two).
$endgroup$
– ΑΘΩ
yesterday

















draft saved

draft discarded
















































Thanks for contributing an answer to Mathematics Stack Exchange!


  • Please be sure to answer the question. Provide details and share your research!

But avoid


  • Asking for help, clarification, or responding to other answers.

  • Making statements based on opinion; back them up with references or personal experience.

Use MathJax to format equations. MathJax reference.


To learn more, see our tips on writing great answers.




draft saved


draft discarded














StackExchange.ready(
function ()
StackExchange.openid.initPostLogin('.new-post-login', 'https%3a%2f%2fmath.stackexchange.com%2fquestions%2f3144880%2fproblem-from-dixonmortimer-textbook-allegedly-sufficient-conditions-for-a-tr%23new-answer', 'question_page');

);

Post as a guest















Required, but never shown





















































Required, but never shown














Required, but never shown












Required, but never shown







Required, but never shown

































Required, but never shown














Required, but never shown












Required, but never shown







Required, but never shown







Popular posts from this blog

Lowndes Grove History Architecture References Navigation menu32°48′6″N 79°57′58″W / 32.80167°N 79.96611°W / 32.80167; -79.9661132°48′6″N 79°57′58″W / 32.80167°N 79.96611°W / 32.80167; -79.9661178002500"National Register Information System"Historic houses of South Carolina"Lowndes Grove""+32° 48' 6.00", −79° 57' 58.00""Lowndes Grove, Charleston County (260 St. Margaret St., Charleston)""Lowndes Grove"The Charleston ExpositionIt Happened in South Carolina"Lowndes Grove (House), Saint Margaret Street & Sixth Avenue, Charleston, Charleston County, SC(Photographs)"Plantations of the Carolina Low Countrye

random experiment with two different functions on unit interval Announcing the arrival of Valued Associate #679: Cesar Manara Planned maintenance scheduled April 23, 2019 at 00:00UTC (8:00pm US/Eastern)Random variable and probability space notionsRandom Walk with EdgesFinding functions where the increase over a random interval is Poisson distributedNumber of days until dayCan an observed event in fact be of zero probability?Unit random processmodels of coins and uniform distributionHow to get the number of successes given $n$ trials , probability $P$ and a random variable $X$Absorbing Markov chain in a computer. Is “almost every” turned into always convergence in computer executions?Stopped random walk is not uniformly integrable

How should I support this large drywall patch? Planned maintenance scheduled April 23, 2019 at 00:00UTC (8:00pm US/Eastern) Announcing the arrival of Valued Associate #679: Cesar Manara Unicorn Meta Zoo #1: Why another podcast?How do I cover large gaps in drywall?How do I keep drywall around a patch from crumbling?Can I glue a second layer of drywall?How to patch long strip on drywall?Large drywall patch: how to avoid bulging seams?Drywall Mesh Patch vs. Bulge? To remove or not to remove?How to fix this drywall job?Prep drywall before backsplashWhat's the best way to fix this horrible drywall patch job?Drywall patching using 3M Patch Plus Primer